LSAT and Law School Admissions Forum

Get expert LSAT preparation and law school admissions advice from PowerScore Test Preparation.

 Administrator
PowerScore Staff
  • PowerScore Staff
  • Posts: 8916
  • Joined: Feb 02, 2011
|
#33309
Complete Question Explanation

Flaw in the Reasoning—CE. The correct answer choice is (D)

This author presents the results of a study of two groups of people—one group of people who had recently developed cancer, and one group without cancer. During the five years leading up to the study, the two groups consumed similar amounts of yogurt, which contains galactose, a substance processed by an enzyme in the body.

The group with cancer did not have sufficient levels of the enzyme to process the galactose in the yogurt they ate, so the author concludes that excessive galactose causes cancer:
  • Premise: ..... Those with cancer were the ones who were unable to process all of the galactose they consumed.

    Conclusion: ..... Galactose in excess of the body’s processing ability causes cancer.
The author’s argument is somewhat questionable, in that he or she notes a correlation between the inability to process galactose and the presence of cancer, and concludes that the excess galactose must be the cause of the cancer.

The question that follows the stimulus asks for the answer choice that weakens the author’s argument. The correct answer choice will undermine the author’s causal conclusion (that the excess galactose is the cause, and cancer is the effect).

Answer choice (A): There is no way to maintain perfect controls on every factor—the study was centered around the body’s absorption of galactose, and the dietary habits of the people do not need to have been the same in all other respects. Since this is not the causal flaw reflected in the stimulus, this cannot be the right answer to this Flaw question.

Answer choice (B): The recommendation in this choice is based on the author’s questionable conclusion that excess galactose is carcinogenic. Even if that conclusion were not questionable, however, the argument would not require that such a recommendation be made. In any case, this is not the causal flaw prephrased above, so this is not the flaw in the author’s reasoning.

Answer choice (C): The fact that other carcinogenic substances exist is not relevant to the argument in the stimulus; focusing on only one such substance is a common approach—it is not a reasoning flaw, so this choice should be eliminated from contention.

Answer choice (D): This is the correct answer choice. The author jumps to the conclusion that excess galactose must cause cancer:
  • Cause ..... ..... ..... Effect

    Galactose ..... :arrow: ..... Cancer
…without considering the possibility that cancer leads to low enzyme levels (and correspondingly excessive galactose levels):
  • Cause ..... ..... ..... Effect

    Cancer ..... :arrow: ..... Galactose
Answer choice (E): The focus is on the fact that for the cancer patients, the enzyme level was insufficient to process the galactose they were consuming; there is no need to specify whether anyone lacked the enzyme entirely, so this choice does not describe the flaw in the author’s reasoning.
 SherryZ
  • Posts: 124
  • Joined: Oct 06, 2013
|
#11775
Thank you for answering my question in advance!

June 2013 LSAT Sec 1, Q3:

I picked answer choice D, which is correct. However, I found that A is very attractive when I was doing the question. Could you explain Why A is NOT correct?


Thanks again!

Sincerely,
Sherry
 Steve Stein
PowerScore Staff
  • PowerScore Staff
  • Posts: 1153
  • Joined: Apr 11, 2011
|
#11778
Hi Sherry,

It's great that you got the right answer to that one--a lot of students find that one tricky, particularly considering that it showed up so early in the section.

Answer choice (A) suggests that the argument fails to consider whether the two groups' diets were the same in every other respect. This is not an issue though, as the study was centered around the body’s absorption of galactose, so the dietary habits of the people do not need to have been the same in all other respects.

I hope that's helpful! Please let me know whether this is clear.

Thanks!

~Steve
 SherryZ
  • Posts: 124
  • Joined: Oct 06, 2013
|
#11780
Hi Steve,

Thank you for replying my question so quickly! I really appreciate it! :lol:

I found A is very attractive when I was doing the question is because that A seems can bring up other possibilities which could cancer, therefore, could hurt the conclusion that too much galactose cause the cancer.

Could you explain further about the flaw of my thought above?

Thanks again!

--Sherry
 Steve Stein
PowerScore Staff
  • PowerScore Staff
  • Posts: 1153
  • Joined: Apr 11, 2011
|
#11789
Hi Sherry,

Thanks for your response. Again, as you know (based on the fact that you got that one right) the author notes a correlation between excess galactose and cancer, and draws a causal conclusion--that is the flaw.

But there is no way to control for every factor; the argument doesn't consider whether everyone in the groups had the exact same diet, but it didn't need to--nor did it need consider whether they all lived at the same altitude, practiced the same exercise habits, or had access to the same level of health care. Had the answer explicitly referred to an alternative cause, that would be different, but the simple failure to consider that the groups might have been eating different things is not the flaw in this case.

I hope that's helpful! This can be a tricky concept, so please let me know whether this is clear--thanks!

~Steve
 SherryZ
  • Posts: 124
  • Joined: Oct 06, 2013
|
#11790
Dear Steve,

Thank you for your reply!

I took the online course and I learned that in order to weaken argument, we can bring up new possibilities, or when identify causal conclusion, we can attack it by:

a. Find an alternate cause for the stated effect
b. Show that even the cause occurs, the effect does not occur.
c. Show that although the effect occurs, the cause did not occur.
d. Show that the stated relationship is reversed
e. Show that a statistical problem exists with the data used to make the causal statement.

But when I saw this question, I thought that A could bring up new possibilities (if ppl of two groups eat different things, then too much galactose might not be the reason causes cancer); At the same time, I found that D can weaken the argument by reversing the relation.

Do you have any suggestion about this situation? Thanks a lot!

---Sherry
 Steve Stein
PowerScore Staff
  • PowerScore Staff
  • Posts: 1153
  • Joined: Apr 11, 2011
|
#11796
Thanks for your response,

For others reading, the right answer to this Causal Flaw question is (D): it explicitly says that the author overlooks another causal possibility:

  • ..... Cause ..... :arrow: ..... Effect

    ..... Cancer ..... :arrow: ..... low enzyme levels

(A) is different from (D), in that it does not state that there is another possible cause; instead, it makes the bold claim that the author should have considered whether everyone in the study had the exact same diet-- and that failing to do so invalidates the whole argument.

I hope that's helpful! Let me know--thanks!

~Steve
 kcho10
  • Posts: 69
  • Joined: Nov 02, 2015
|
#35642
Hello,

For causal arguments, isn't one of the main assumptions that the stated cause is the ONLY cause? And wouldn't C thereby weaken that assumption?
 Kristina Moen
PowerScore Staff
  • PowerScore Staff
  • Posts: 230
  • Joined: Nov 17, 2016
|
#35840
Hi kcho,

In causal arguments, there is no assumption that it is the ONLY cause (unless stated). You can still weaken an argument by showing that there is another cause. Remember that a Weaken question does not ask you to KILL an argument, but to weaken it at least a little bit. You are driving a wedge between the premises and the conclusion. So while it's not an assumption of the argument that it's the only cause (it could still be a cause, even if it's not the only one), you drive a wedge in the causal relationship by giving an alternate cause.

Here, you are looking for the flaw. You are not looking just to weaken the argument, but to point out a fundamental flaw. And one flaw here is that the cause and effect could be reversed. That effectively KILLS the causal argument. Remember there is a time element - the cause must come BEFORE the effect. So there is no way for something to be both the effect AND the cause.

However, it is possible for an effect to have more than one cause. It merely weaken a causal argument to show there's a second or third cause.

Hope this helps.

- Kristina
 Mariam
  • Posts: 21
  • Joined: Apr 04, 2020
|
#80090
Hello-
I understand why D is the correct answer choice. Can someone please elaborate on why A is incorrect? I thought D was a much better choice but A suggests an alternative explanation.

Thanks

Get the most out of your LSAT Prep Plus subscription.

Analyze and track your performance with our Testing and Analytics Package.